Pandora's box

Find the smallest whole number that, when divided by 5, 7, 9, and 11, gives remainders of 1, 2, 3, and 4, respectively.


The answer is 1731.

This section requires Javascript.
You are seeing this because something didn't load right. We suggest you, (a) try refreshing the page, (b) enabling javascript if it is disabled on your browser and, finally, (c) loading the non-javascript version of this page . We're sorry about the hassle.

5 solutions

Adriana Batalha
Aug 18, 2015

Let n n be the number we are trying to find. We know that

5 n + 4 5|n+4 \Rightarrow 5 2 n + 8 5|2n+8 \Rightarrow 5 2 n + 3 5|2n+3

7 n + 5 7|n+5 \Rightarrow 7 2 n + 10 7|2n+10 \Rightarrow 7 2 n + 3 7|2n+3

9 n + 6 9|n+6 \Rightarrow 9 2 n + 12 9|2n+12 \Rightarrow 9 2 n + 3 9|2n+3

11 n + 7 11|n+7 \Rightarrow 11 2 n + 14 11|2n+14 \Rightarrow 11 2 n + 3 11|2n+3

Since 5, 7, 9 and 11 are co-primes, the smallest number that is divisible by all of them is 5 × 7 × 9 × 11 = 3465 5 \times 7 \times 9 \times 11 = 3465

2 n + 3 = 3465 2n+3 = 3465 \Rightarrow n = 1731 n = 1731

Arturo Presa
Nov 25, 2016

We need to find the smallest number x x such that x 1 ( m o d 5 ) , x 2 ( m o d 7 ) , x 3 ( m o d 9 ) , x\equiv 1\pmod 5,x\equiv 2\pmod 7, x\equiv 3\pmod 9, and x 4 ( m o d 11 ) . x\equiv 4\pmod {11}. Let y 1 = 7 9 11 = 693 3 ( m o d 5 ) , y_1= 7*9*11 = 693\equiv 3 \pmod 5, y 2 = 5 9 11 = 495 5 ( m o d 7 ) , y_2=5*9*11 =495 \equiv 5 \pmod 7, y 3 = 5 7 11 = 385 7 ( m o d 9 ) , y_3= 5*7*11 = 385 \equiv 7 \pmod 9, and y 4 = 5 7 9 = 315 7 ( m o d 5 ) . y_4= 5*7*9 = 315 \equiv 7 \pmod 5.

Then, according to the proof of Chinese Remainder Theorem a solution x x of the simultaneous congruences above can be obtained in the form x = 1 y 1 x 1 + 2 y 2 x 2 + 3 y 3 x 3 + 4 y 4 x 4 , x=1*y_1*x_1+2*y_2*x_2+3*y_3*x_3+4*y_4*x_4, where y 1 x 1 , y 2 x 2 , y 3 x 3 , y_1*x_1, y_2*x_2, y_3*x_3, and y 4 x 4 y_4*x_4 are congruent to 1 modulus 5, 7, 9, and 11, respectively. There is a systematic way of finding x 1 , x 2 , x 3 , x_1, x_2, x_3, and x 4 x_4 by using the Euclid extended algorithm, but in this case we can find them just by trial and errors. For example, since y 1 3 ( m o d 5 ) , y_1\equiv 3 \pmod 5, then 3 x 1 ( m o d 5 ) 3*x_1\equiv \pmod 5 , replacing x 1 x_1 by 1, 2, 3, and 4, we get that the value that solves the latter congruence is 2. Similarly, we obtain that x 2 = 3 , x 3 = 4 , x 4 = 8. x_2=3, x_3=4, x_4=8.

So, according to the expression of x x that we mention above, we obtain that x = 1 693 2 + 2 495 3 + 3 385 4 + 4 315 8 = 19056. x=1*693*2+2*495*3+3*385*4+4*315*8=19056. Then according to the Chinese Remainder Theorem, any solution of the given congruences must be congruent to 19056 modulus 5 7 9 11 = 3465. 5*7*9*11=3465. The smallest number satisfying this condition is the remainder of the division of 19056 by the 3465,which is 1731.

You've made a typo while calculating y4.

Akash S M - 2 years, 10 months ago
Garrett Clarke
Jun 9, 2015

Start by searching for numbers that give you a remainder of 1 when divided by 5. These are numbers that end in either 1 or 6. Then, try to find numbers that end in 1 or 6 that give you a remainder of 2 when you divide by 7. The first number of this kind turns out to be 16. In order to find other numbers that satisfy this property, notice that if X = 16 + ( 5 × 7 ) × n X=16+(5\times7)\times n , dividing by 5 still leaves remainder 1 and dividing by 7 still leaves remainder 2. This means if we keep adding by 5 × 7 = 35 5\times7 = 35 , our first two properties are maintained. Now, search for a number that satisfies the first 3 properties using this method. The first number that works is 16 + 35 × 4 = 156 16+35\times4 = 156 . Again we find the pattern, AKA then number we can add by to maintain our properties. Let X = 156 + ( 5 × 7 × 9 ) × n X = 156+(5\times7\times9)\times n and as you can see the properties hold. Now add 5 × 7 × 9 = 315 5\times7\times9=315 to 156 continuously to find the first instance of a number that works with your fourth and final property. The first value that comes up is 156 + 315 × 5 = 1731 156+315\times 5=1731 , giving us our final answer.

why is the multiple sign in 16+(35x4)=156 gone? and so on in the remaining numbers, this solution is very confusing, [Now add 579=315 to 156]? [X=16+(57)n], shouldnt it be x=16+(5x7)n

Hau You - 6 years ago

Log in to reply

Sorry about that Hau You, I wrote this while I was in the process of learning how to use Latex to write answers so some things became a bit garbled in the solution. I've edited my answer, it should make a bit more sense now!

Garrett Clarke - 6 years ago
Ankur Sharma
May 10, 2017

Let number is X. X=11k+4=3(mod9) So k=9a+4 X=11(9a+4)+4=99a+48 Now X=99a+48=2(mod7) a=7b+3 Put value of a we will get X=693b+345=1(mod5) So here b=2 for smallest number X=693×2+345=1731

Oscar Rojas
May 14, 2015

X = 1 mod 5

X = 2 mod 7

X = 3 mod 9

X = 4 mod 11

The Chinese remainder theorem, we will get this 19056=1731 mod 3465 the smallest number is 1731.

Moderator note:

Since Chinese Remainder Theorem is the most conventional approach for this kind of problem, can you write out a full solution?

Bonus question : Can you think of another way that doesn't implore Chinese Remainder Theorem?

how does this work actually? how to get 19056? pls help tq

JOHNNY CHIN - 4 years, 11 months ago

Chck for 191

Shiv Tavker - 6 years ago

Log in to reply

no its wrong it doesn't gives 3 rem. when div by 9 @Shiv Tavker

Harshi Singh - 5 years, 10 months ago

Can anyone send me the full solution by using Chinese remainder theorem

Prabhat Rao - 4 years, 9 months ago

0 pending reports

×

Problem Loading...

Note Loading...

Set Loading...